Professional Responsibility Final

¡Supera tus tareas y exámenes ahora con Quizwiz!

Among the four categories of wrongdoing listed below, which is most likely to result in a lawyer being disbarred permanently? Stealing money from a client. Not returning clients' telephone calls. Not responding to discovery requests on time. An arrest for driving while intoxicated.

Stealing money from a client.

True or false: In the attorney-client context, the concept of confidentiality has both an evidentiary basis and a common-law basis reflected in the fiduciary duty lawyers owe clients.

True

True or false: It is a good practice for a lawyer to send a nonengagement letter when declining to represent a prospective client.

True

True or false: The first step in assessing a potential conflict of interest is identifying the client.

True

True or false: When a client hires a lawyer, it is comparable to "at-will" employment and the client may terminate the representation for any reason.

True

True or false: When reviewing bar applications, bar examiners are more likely to be concerned with nondisclosure of a petty misdemeanor citation than the citation itself.

True

True or false: Conflicts of interest arise only in litigation or when litigation is anticipated.

False

True or false: In a large corporation where employees receive stock as part of their compensation, the corporation's general counsel owes fiduciary and loyalty duties to the employees and corporation equally because the employees own part of the corporation.

False

True or false: In pending civil litigation in federal court, if counsel for a plaintiff and defendant agree the plaintiff's lawyer should withdraw from representing the plaintiff, a judge is always bound by the lawyers' intentions.

False

True or false: It is the client's responsibility to ask whatever questions are necessary for the client to make substantive decisions about a matter.

False

True or false: The general rule is that a mediator cannot provide legal advice to the parties in a mediation. However, an exception exists for unrepresented parties.

False

True or false: Under the Model Rules, a lawyer with a side interest in gourmet cooking may start a second business with a chef and operate a restaurant as a for-profit partnership, with the lawyer and chef splitting the fees.

False

True or false: When a litigation client fails to pay the lawyer's fee, nonpayment is a material breach of the lawyer-client agreement and the lawyer may always terminate representation without approval from the court.

False

When a lawyer has a dispute over fees with a client, which among the following is the best practice to follow? A. Arbitrate or mediate the dispute if required, and keep the disputed amount in a trust account. B. Commence a declaratory judgment action or interpleader action against the client to obtain a judicial determination of the right to the property. C. Send the client a check for the amount the lawyer reasonably believes the client is due, and rely on the doctrine of "accord and satisfaction" if the client cashes the check. D. Stop all work on the matter until the dispute is resolved.

A. Arbitrate or mediate the dispute if requred, and keep the disputed amount in a trust account.

You have been at a medium-sized litigation law firm for two years. Your mentor, who has built a 25-year relationship with Farmstate, Inc., an insurance company with a catchy advertising jingle, has entrusted you with your first case in which the damages could exceed $500,000. A construction contractor, Builditwrong Inc., has been sued for defective construction. Your mentor has taken care of doing the conflicts check and sending engagement letters. As you know from being copied on her various emails, Farmstate is paying the legal fees to defend the lawsuit. You call to introduce yourself to the CEO of Builditwrong and provide your email address. Unprompted, the CEO sends you a copy of the insurance policy issued by Farmstate, Inc. At that point, and applying the principles of Pine Island Farmers Coop. v. Erstad & Riemer, who is your client? A. Builditwrong, Inc., because the corporation is the insured. B. Farmstate, Inc., because by sending the insurance policy Builditwrong has impliedly consented to a dual defense. C. Both of the above. D. Neither of the above.

A. Builditwrong, Inc., because the corporation is the insured.

According to the Model Rules of Professional Conduct, a contingent-fee arrangement is permitted for which of the following type or types of representation? A. Defense against civil claims, as long as the amount is reasonable. B. Representation of a plaintiff in a personal-injury action, as long as the lawyer's website provides written explanation for how the fee will be computed. C. Answered Both of the above. D. Neither of the above.

A. Defense against civil claims, as long as the amount is reasonable.

You are one of four lawyers at a criminal-defense law firm. You work half-time on "fee-paying work" and half time for the Public Defender's Office under a contract through which you provide defense to defendants who cannot afford lawyers. In your role as a public defender, the court has appointed you to jointly represent Robin and Greta in a convenience-store robbery. During their initial appearances in court, you make sure each client's rights are protected as each client pleads not guilty. The next day, you visit the jail to speak with your clients for the first time. Although the jailer makes a fuss when you say you want to meet with each client individually, eventually the jailer agrees. You meet with Greta first. She tells you Robin was the mastermind behind it all and Greta was "just the getaway driver." As proof, Greta said she is prepared to show you an iPhone selfie video she took while waiting in the car. According to Greta, Robin was inside the store when she made the video and he returned to the car shortly after Greta stopped the video. She says she cannot show you the video now because a sheriff's deputy confiscated her phone when she was booked into the jail. Based on the assigned rules and reading and the week's office hour, what is your best course of action? A. End the interview and prepare paperwork to withdraw from representation of both Robin and Greta. B. Continue the interview, get as much information as possible, ask the jailer to provide you access to the phone, and show it to Robin to get his reaction. C. End the interview, speak with Robin, and prepare to withdraw from representing him if his story conflicts with Greta's. D. Continue the interview, and then explain to Robin why someone else at your law firm will have to represent him.

A. End the interview and prepare paperwork to withdraw from representation of both Robin and Greta.

A college friend, Dale, calls you early one morning asking if you can help with a delicate legal situation. You take down just enough information to determine there is no conflict of interest and then agree to listen. You learn Dale's 22-year-old daughter, Rebecca, has been acting erratically the past few months after losing her job. Late yesterday afternoon, Rebecca was observed loading computers belonging to her former employer into her car. Dale provides further details about Rebecca's erratic behavior, and although you are not a psychiatrist the acts sound like potential schizophrenia. No acts have been violent. Dale knows one of the co-owners of the business from which Rebecca was fired. The co-owner calls Dale and threatens to press charges unless Rebecca returns the property by noon today. Dale asks you to help and is willing to pay your hourly rate. You set up an initial consultation. Who should attend and why? A. Only Rebecca, because she is the client. B. Rebecca and Dale, because it is reasonable to expect a young adult will want a parent involved. C. Rebecca, Dale, and the former employer's co-owner, because this matter seems easily resolvable. D. Only Dale, because Rebecca has diminished capacity.

A. Only Rebecca, because she is the client.

n law school, Pedro and Dana were friends but then went separate ways, Pedro to become a county prosecutor and Dana to work as a criminal defense attorney. They remained good friends even though they were careful not to discuss client confidences. After an Association of Criminal Attorneys evening happy hour, Pedro drank only Coke and offered to drive Dana home. On the way, Pedro received a call. The vehicle's dashboard screen displayed the source of the call as Pedro's supervisor, calling from her county-issued cell phone. Pedro, being a law-abiding prosecutor, was careful not to violate the state statute that prohibited use of handheld devices while driving. So he answered the call in hands-free mode, for both Pedro and Dana to hear. Pedro's supervisor said a "no-knock" search warrant would be executed at 2:00 a.m., or in just a few hours, at the personal residence of "that loathsome Lordsome character." Pedro's supervisor was referring to Terrence Lordsome, a notoriously terrible slumlord in town who was represented by Dana. Pedro and Dana stay silent for the rest of the ride. Upon returning home, Dana calls Lordsome to tell him about the soon-to-be executed warrant. According to the week's reading and office hour, who if anyone has violated a Rule of Professional Conduct and why? A. Pedro, because he failed to take reasonable steps to safeguard confidential client information. B. Dana, because there is a duty to notify the sender of such inadvertently disclosed information. C. Both Pedro and Dana, for the reasons above. D. Neither Pedro nor Dana, becuase Pedro is a prosecutor and does not represent clients

A. Pedro, because he failed to take reasonable steps to safeguard confidential client information.

Sal is a lawyer at a law firm that specializes in real estate matters. Sal's practice is half transactional and half litigation. Last month, he represented Sella in the sale of her home. At the closing (the meeting where the sale is finalized), the buyer was represented by her lawyer. No one else was present. As it turned out, the home suffered from leaky windows. The buyer alleged both Sal and Sella made misstatements of fact during the closing about the home being dry and the windows being high-quality. The buyer retains new legal counsel to rescind the transaction, alleging the statements were fraudulent. May Amos, another lawyer at Sal's law firm, defend Sella in the litigation consistent with the Model Rules of Professional Conduct and office-hour discussion? A. Yes, as long as Amos has no conflict of interest. B. Yes, because the crime-fraud exception applies. C. No, because Sal will be a necessary witness. D. No, because the attorney-client privilege applies.

A. Yes, as long as Amos has no conflict of interest.

Disillusioned with both criminal defense and big-city litigation, Mario loaded his Mazda Miata to the gills and headed to a small town on the Vermont-New York border, to get away from it all. While scouring the town's two grocery stores for organic toilet paper, Mario struck up a conversation with Doris, one of two attorneys in town, who invited him to coffee. Mario suggested a meeting over Zoom instead because he had become so accustomed to social distancing, but Doris insisted on an in-person chat. They hit it off and agreed Mario would ease his way into Doris' practice, and eventually buy it. Most of the practice, with an office in New York State, involved estate planning, real estate work, and a little work with local vineyards. Doris was licensed in both New York and Vermont. Mario liked his wine, so to start with Doris and Mario agreed he would focus his practice only on the vineyard clients, work hard to grow it, and pay one-third his salary toward the firm's $200,000 purchase price. According to the plan, Doris would handle the rest of the matters, but from Key West for six months of the year, while easing Mario into the estate planning and real-estate work gradually. They figured Mario would complete purchase of the firm in three years. They did not discuss whether Mario would also be licensed in Vermont, but they did change the firm's name to incorporate both their last names. Based on the Rules of Professional Conduct, the week's reading, and our office-hour discussion, it their arrangement permissible? A. Yes, as long as Mario focuses his practice on matters implicating New York law and associates closely with Doris on a limited number of cases implicating Vermont law. B. Yes, because Doris has a property interest in her law practice and has a legal right to convey it on an installment plan. C. No, because Mario needs to buy the whole practice and find other means to finance it if he cannot buy it outright. D. No, because Doris cannot supervise Mario from Florida.

A. Yes, as long as Mario focuses his practice on matters implicating New York law and associates closely with Doris on a limited number of cases implicating Vermont law.

Corinne helps Delilah negotiate a sales price of $6.2 million, a relative bargain! Delilah and Delmar bring a cashier's check for the full amount to the law office. Delilah does the talking and explains they want Devin to have the property upon their death. Delilah directs Corinne to prepare a warranty deed for the sellers' signature that will list Delilah, Delmar, and Devin as "joint tenants." Delmar stays silent and Devin is not there. Corinne asks about Devin's absence, and Delilah explains Devin is studying for the LSAT. Plus, according to Delilah, the couple want Devin to be "surprised" someday when he receives full title to the property. Because Delilah has bought and sold lots of real estate, she knows that under a joint tenancy the surviving tenant acquires the property upon death of the other tenants. Corinne knows Delilah is familiar with joint tenancies because the lawyer has represented Delilah in similar transactions. Delmar asks no questions. The sale is completed and the deed is signed and recorded. Corinne emails a pdf to Delilah. Sadly, just a week after the sale, Delilah and Delmar die in a tragic accident involving motorized scooters on St. Paul's Grand Avenue. Devin is upset, not only about the deaths but by news that he is now responsible for $142,000 a year in property taxes on his new property, plus another $24,000 for property insurance. He has no idea how he can pay the taxes and also law school tuition. He writes a text message to Corinne to complain. Based on the week's reading and office hour with Kelly Jeanetta, did Corinne violate provisions including the Rules of Professional Conduct by keeping the Richies' "surprise"? A. Yes, because Corinne should have anticipated Devin could someday incur substantial tax liability and communicated with him accordingly. B. Yes, because Corinne had actual knowledge the Richies were older than 65 and she had a duty to foresee they might have died soon. C. No, because Corinne had a contractual and professional obligation to keep the couple's "surprise." D. No, because Devin can sell the property and come out way ahead.

A. Yes, because Corinne should have anticipated Devin could someday incur substantial tax liability and communicated with him accordingly.

Gunner works as an associate in the Entertainment Law Group at Huge, Big, & Bigger, a law firm with nearly 1,000 lawyers in 30 offices around the world. Gunner is in the Minneapolis office, which has 10 lawyers. Besides billing 2,500 hours a year, Gunner is expected to demonstrate "business acumen," which means finding and retaining clients willing to pay rates ranging from $400 to $1,500 an hour. Gunner trolled her phone's contacts for potential clients and got a brainstorm. Why not invite Sean to lunch? Gunner and Sean were in a band together in college. Sean majored in music and enjoyed moderate success as a solo artist. Over lunch, Sean explained to Gunner he was having a dispute with a record label. Gunner said maybe she could help. Sean asked about Gunner's hourly rate. She said the rate was $450 an hour, but maybe she could lower it. Gunner explained she needed her supervisor's approval to do so. Upon hearing that, Sean said he had done preliminary legal research on the internet and believed he was owed $10,000 for copyright infringement. Sean said he would take $2,500 to settle the matter. Sean also said he was aware of at least 20 other artists with similar complaints about the record label who had also expressed willingness to settle. He wrote down a half-dozen names and gave them to Gunner. Back at the office, Gunner could hardly contain her excitement when telling the Minneapolis managing partner about bringing in what appeared to be her first client. It sounded like a case of clear-cut copyright infringement, she said, with many more clients waiting in the wings. The managing partner's first question was, "Which record label?" Gunner said she did not know. The managing partner told her to find out. As it turned out, a partner at Huge, Big & Bigger's Los Angeles office represents the label in pending copyright-infringement litigation involving the record label's alleged fraudulent tactics in obtaining works by lesser-known bands. The law firm has received nearly $1 million in legal fees to defend the record label. Based materials we studied this week and earlier in the course, is Huge, Big & Bigger at risk of being disqualified as counsel for the record label? A. Yes, because Gunner did not obtain the name of the record label before hearing about the merits of Sean's dispute. B. Yes, because a conflict of this type is non-consentable. C. No, as long as Gunner is screened from all files from the Los Angeles office and the firm provides notice to the record label. D. No, because Sean never became a client.

A. Yes, because Gunner did not obtain the name of the record label before hearing about the merits of Sean's dispute.

Wanda is an attorney who worked in the Whisler County Attorney's Office for 20 years in a variety of positions ranging from criminal prosecution to the Real Estate Division. She had glowing job reviews. Her latest position was in the Civil Litigation Division, where she defended civil claims against agencies including the Highway Department. Last month, she uncovered evidence that private road-construction contractors were paying kickbacks to elected members of the County Board. Wanda asked for a private meeting with her supervisor to share what she knew, but the supervisor did nothing. Therefore, the next week, Wanda emailed some but not all of the information to her supervisor and cc'd the supervisor's loose-lipped assistant, knowing there was a good chance the email would be leaked to the news media. It was, in fact, leaked. As a result, the Chair of the County Board succeeded in having Wanda fired for not following protocol. Based on the week's readings and office hours, is there a possibility Wanda will be permitted to disclose the contents of what was said during the private meeting with her supervisor? A. Yes, because Wanda has a right to defend herself to demonstrate she was fired for pointing out wrongdoing. B. Yes, because the attorney-client privilege is inapplicable in the government context. C. No, because the information from the private meeting was not in a tangible form and litigation was not being anticipated. D. No, because the information is protected by the attorney-client privilege and there is nothing to indicate the county, who is the client, waived the privilege.

A. Yes, because Wanda has a right to defend herself to demonstrate she was fired for pointing out wrongdoing.

The matter of Abel v. Bain settles. The parties sign mutual releases agreeing they have no current or future claims against each other and receive disengagement letters. Liang leaves Furst & Furst to join Seconda Law, Ltd. She brings her file in Abel v. Bain with her and keeps the file in her office. Among Liang's new law partners at Seconda Law is Newton. Bain calls Newton asking Newton to represent Bain in an employment dispute with his employer, BadCo. May Newton do so consistent with the Model Rules of Professional Conduct and comments, class video, and office hour? A. Yes, because based on this information Bain's employment dispute has nothing to do with Bain's car accident. B. Yes, because a valid settlement agreement was in effect. C. No, because Seconda Law is essentially one lawyer. D. No, because Liang brought her file in Abel v. Bain and has significantly harmful information about Bain.

A. Yes, because based on this information Bain's employment dispute has nothing to do with Bain's car accident.

Sona is a licensed attorney whose office is in Schaumburg, Illinois, a western suburb of Chicago. Her billable-hour rate is $400 an hour. Sona represents a client who has a motion-to-dismiss hearing in Bloomington, Illinois, a two-hour drive to the south. In the engagement letter, which the client signed, the client agreed to be billed hourly, in six-minute increments. The client also agreed to pay for Sona's travel time. On her way from Schaumburg to Bloomington, Sona spends a half-hour in a hands-free conference call in another matter, a half-hour rehearsing her oral argument, and an hour listening to a podcast about running half-marathons. According to the Model Rules, the week's reading, and the office hour, how much may Sona bill the client for the time spent driving from Schaumburg to Bloomington? A. $800, because the trip from Schaumburg to Bloomington takes two hours. B. $600, because Sona spent a half-hour working on an unrelated matter. C. $200, because Sona should have been practcing her oral argument for the whole two hours. D. Only the cost of mileage, because any windfall goes to the client.

B. $600, because Sona spent a half-hour working on an unrelated matter

The election for an open seat on the Smallville City Council was contentious. There were four candidates for one open seat. A key issue was whether a city ordinance should be amended to allow homeless persons to live in city parks temporarily during the pandemic. B. Javier Badly, a local licensed attorney, campaigned against the ordinance as a "health risk." Proponents of the ordinance accused Badly of discriminating against homeless people, which he denied. Rather, after Badly lost, he used social media to make unsupported allegations of "elections fraud" against the Smallville Elections Manager. It got so ugly a county judge issued an order prohibiting acts of harassment against the Elections Manager. Attorney Badly believed the order was unconstitutional. He directed his law clerk Iris Green, a first-year law student, to organize a noisy protest at the Elections Manager's home at 2 a.m. and to direct chants toward the Elections Manager by name. When Iris did so, she was arrested for disorderly conduct. During a police interview (preceded by a Miranda warning), Iris said her boss, Attorney Badly, told her to organize the protest. In court the next week, Iris stated under oath she did not know about the judge's order. She agreed to an arrangement by which the charge would be dismissed if she committed no same-or-similar offenses for two years. Attorney Badly was not charged with a crime. Under the Model Rules of Professional Conduct, whose right to practice law is at risk and why? A. Attorney Badly, because he took a campaign position that discriminates against persons based on socioeconomic status. B. Attorney Badly, because he used Iris as a surrogate to violate a court order. C. Iris, because her plea deal was basically an admission of guilt and criminal convictions prevent bar applicants from obtaining law licenses as a matter of law. D. Both Badly and Iris, for the reasons above.

B. Attorney Badly, because he used Iris as a surrougate to violate a court order.

Your work on Rebecca's situation could not be resolved in just a few hours because Rebecca says the computer equipment was a gift to her from her former employer. The idea seems implausible to you, but the bottom line is the day ends without Rebecca returning the property. Two days later, a lawyer who says she represents Rebecca's former employer calls Dale, who provides the lawyer with your name and number. After confirming you represent Rebecca, the lawyer says that if Rebecca is willing to pay $10,000 (the market value of the computer equipment), the employer will not "press charges." What do you do? A. Seek a court-ordered mental-health evaluation because the implausibility of Rebecca's position only adds to her diminished capacity. B. Call Rebecca to ask whether she is willing to pay $10,000. C. Call Dale to ask whether he might be able to transfer $10,000 into Rebecca's bank account. D. End the call, determine from eBay the market value of the computers is $5,000, and call the lawyer back with a $5,000 counteroffer.

B. Call Rebecca to ask whether she is willing to pay $10,000.

You represent George, among family members embroiled in a dispute about control of a family business. You are paid on an hourly basis (not on a contingent-fee arrangement). George has sued his twin brother Gus, alleging various business torts. You describe the typical course of litigation of that type, including that it is customary to take depositions. When George asks you how much depositions cost, you say it it depends on how long the deposition goes but generally a four-hour deposition will cost around $3,000 in attorney time and court-reporter fees. George bristles at the idea and says you may not spend more than $10,000 in depositions. Based on the week's assigned reading and office hour with Professor Jeanetta, which among the following is your best course of conduct? A. Take as many depositions as you need and pay for them yourself if necessary. B. Discuss the ramifications of limiting the number of depositions, document the discussion in an email to George, and follow George's directive. C. Negotiate a flat-fee arrangement with the court reporter. D. Make inquiries about George's potential diminished capacity.

B. Discuss the ramifications of limiting the number of depositions, document the discussion in an email to George, and follow George's directive.

Based on Model Rules and week's reading, which of the following is not a factor for determining whether an attorney's fee is reasonable? A. How much lawyers in the community typically charge for the specific type of matter. B. How much the client could be forced to pay. C. Whether the matter prevented the lawyer from representing other clients. D. The lawyer's level of experience.

B. How much the client could be forced to pay.

Wayne is an associate attorney at a law firm that specializes in Fair Labor Standards Act (FSLA) litigation. This includes civil actions alleging employers do not pay employees overtime when they should. The litigation can be lucrative for plaintiffs' attorneys because a statute allows for prevailing plaintiffs to recover their attorneys' fees.Wayne's law firm has an FSLA case pending in which LotsaCall, Inc. is the Defendant and is represented by counsel. LotsaCall is a call center where employees take calls from around the world 24 hours a day, holidays included. The judge has ordered a status conference on the litigation for next week. To gather facts ahead of the conference, Wayne sets up his laptop computer in a coffee shop across the street from the call-in center with hopes of interviewing persons with knowledge of LotsaCall's practices and procedures. But of course, Wayne wants to act ethically. Based on the week's readings and Model Rules, as well as our discussion during the office hour, which category of persons should Wayne feel most comfortable talking to? A. LotsaCall's managers. B. LotsaCall's lawyers. C. Current employees who take calls. D. You Answered Former employees who took calls.

B. LotsaCall's lawyers.

Which of the following was not identified and discussed as a hindrance to A2J in rural areas? A. Overreliance on technology. B. Lack of judges. C. Conflicts of interest arising from personal, civic, and church-related relationships. D. Lack of "sophisticated" legal work.

D. Lack of "sophisticated" legal work.

True or false: A lawyer should avoid talking about client matters in social settings. However, the common law recognizes an exception for when a social group is composed solely of other lawyers. In that instance, as long as the lawyers are licensed in good standing, the attorney-client privilege applies to any communications among attorneys.

False

You work at a law firm but have represented WerBad, Inc., which manufacturers pogo sticks, for several years. The work has been lucrative for you because the company gets sued in products-liability cases fairly recently. The cases involve consumers who have been injured by the products. The latest lawsuit is a bit unusual because it alleges defective design and names both the corporation and the Vice President for Product Design as co-defendants. From prior work, you know the corporation would pay damages on behalf of the Vice President should she be found liable and her work was in the course and scope of her employment. WerBad's CEO says he stands behind the Vice President "one hundred and ten percent." May you represent both the corporation and Vice President consistent with the week's readings and office hour? A. Yes, because the lawsuit involves multiple defendants, not multiple plaintiffs. B. Most likely, because there is no indication your zealous advocacy of one defendant will materially affect the interests of the other defendant. C. No, because the situation presents an unwaivable conflict of interest. D. No, because you have confidential information from previous lawsuits against the company.

B. Most likely, because there is no indication your zealous advocacy of one defendant will materially affect the interests of the other defendant.

Wayne is taking a deposition in the LotsaCall matter. The deposition witness is Carl, a former employee who has since retired. LotsaCall is represented by Natasha, who takes the position that she represents Carl in the case. Time after time, Natasha orders Carl not to answer Wayne's questions on grounds the questions are "vague" and "irrelevant." When Wayne asks Carl whether Natasha represents him in the case, Natasha objects on grounds the question violates the attorney-client privilege. Wayne perseveres, stating on the record that the fact of whether someone is represented by an attorney is not protected by the attorney-client privilege because the fact is not a communication related to seeking or receiving legal advice. Natasha borders on belligerent in her response, scolding Wayne: "This is not how privilege works in the real world, buddy. Don't talk to me about professionalism and local rules, and forget what you learned in law school. The witness is not answering the question, period. Call the bailiff if you want." Wayne asks for a break to call a more senior attorney at his office for advice on how to deal with Natasha. First, he stops in the restroom. A minute later, Carl is in the restroom too. Unprompted, Carl volunteers to Wayne that Natasha does not represent him and says he never met Natasha until five minutes before the deposition. Who has violated the Model Rules of Professional Conduct? A. Wayne, because he should have anticipated Carl also would use the restroom. B. Natasha, because she stood in the way of Wayne obtaining relevant information. C. Both Wayne and Natasha, for the reasons articulated above. D. Neither Wayne nor Natasha.

B. Natasha, because she stood in the way of Wayne obtaining relevant information.

Sue wants to terminate your representation. Based on the week's readings and office hour, which is a step you should NOT take during the process of withdrawing from the representation of Sue? A. Making a copy of the file (and/or safely archiving electronically materials related to the representation). B. Providing all file materials, except for those containing your mental impressions, which are your property. C. Reiterating Sue's obligation to pay costs according to the fee agreement. D. Informing Sue of the specific date when the statute of limitations would prevent a claim.

B. Providing all file materials, except for those containing your mental impressions, which are your property.

This week, we studied curriculum and had an office-hour discussion centered on access to justice ("A2J") in rural areas. Based on the curriculum and discussion, which Model Rule of Professional Conduct (and its state-rule equivalent) is seen as the greatest hindrance to A2J? A. Rule 1.1, "Competence," because it discourages lawyers from experimenting in new practice areas. B. Rule 5.4, "Professional Independence of a Lawyer," because it hinders innovation and makes legal services too expensive for too many. C. Rule 5.5, "Unauthorized Practice of Law; Multijurisdictional Practice of Law," because it discourages law firms from urban states from providing services in less-urban states. D. Rule 8.5, "Disciplinary Authority; Choice of Law," because it unduly burdens lawyers by making them learn ethical rules in numerous states.

B. Rule 5.4, "Professional Independence of a Lawyer," because it hinders innovation and makes legal services too expensive for too many.

When may an attorney withdraw from a client's representation according to the Model Rules of Professional Conduct? A. When a criminal defendant dismisses the lawyer's professional advice on whether to testify at trial. B. When the client has used the lawyer's services to perpetuate a crime. C. When a judge issues an order for withdrawal sua sponte. D. All of the above.

B. When the client has used the lawyer's services to perpetuate a crime.

As you know, generally a lawyer who represents a client in a matter may not discuss the subject of representation with a person whom the lawyer knows is represented by a lawyer in the matter. But when may such a discussion take place consistent with the Model Rules of Professional Conduct? A. When the represented person consents. B. When the represented person's lawyer consents. C. Either of the above. D. Neither of the above.

B. When the represented person's lawyer consents.

Dominique is in his third year of law practice at a law firm in Omaha, Nebraska that represents clients in business transactions. He is licensed in Nebraska and Iowa. Although he rarely goes to court and has never participated in a trial, he has been admitted to the United States District Court for the District of Nebraska. In addition, he had a one-year clerkship for a federal judge whose courtroom is in Lincoln, Nebraska. Yesterday Fiona, a friend from high school who practices commercial litigation in Chicago, asked Dominique to serve as local counsel in a matter that was filed in Nebraska state court. The client wants the matter removed to the United States District Court for the District of Nebraska. Fiona is licensed in Illinois and not admitted to the Nebraska federal court. Fiona has removed cases to federal court at least a dozen times, but never in Nebraska. She has drafted the notice of removal for Dominique's signature and has asked him to file it in the Nebraska federal court. May Dominique do so consistent with the Model Rules of Professional Conduct? A. Yes, because Dominique is associating with Fiona, who is an experienced litigation attorney. B. Yes, as long as Dominique reviews the notice, researches applicable local rules, arranges for Fiona to seek pro hac vice admission in the District of Nebraska, and remains an active participant in the litigation. C. No, because it is unethical for a lawyer to draft a document for another lawyer's signature. D. No, because Dominque has no trial experience and his prior work as a federal law clerk is irrelevant for assessing his competence.

B. Yes, as long as Dominique reviews the notice, researches applicable local rules, arranges for Fiona to seek pro hac vice admission in the District of Nebraska, and remains an active participant in the litigation.

Lisa is licensed to practice law in a state where the Model Rules of Professional Conduct apply. For the past several years, she has worked as an attorney for the General Services Administration ("GSA"), a federal agency that provides a wide range of services for the government. The GSA employs thousands of lawyers. Last year, on behalf of the GSA, Lisa drafted a contract in which RecycAll, a private recycling firm, agreed to pick up recyclables for several federal buildings in the state. Recently, Lisa has left the GSA to become an in-house attorney at RecycAll. Now, Lisa is negotiating a contract between RecycAll and a trucking firm to pick up recyclables at the federal buildings, in furtherance of GSA's contract with RecycAll. May she do so consistent with the Model Rules of Professional Conduct, comments, and office hour? A. Yes, because there is no adversity between RecycAll and the GSA and Lisa is not "switching sides." B. Yes, as long as Lisa obtains the GSA's informed consent, confirmed in writing. C. No, because Lisa's fiduciary duty is as RecycAll's constitutent and she must retain outside counsel to negotiate the contract on behalf of RecycAll. D. No, because the Federal Rules of Professional Conduct would apply.

B. Yes, as long as Lisa obtains the GSA's informed consent, confirmed in writing.

It is 2041 and you have built a successful criminal defense practice. Your ViRGos (virtual reality goggles) buzz and you put them on to find Xavier, a college friend, on the other end. Xavier's 21-year-old daughter has been arrested for drunken hovering while operating Xavier's hovercar. (A 2041 hovercar like a 2021 car, except it hovers above the ground.) Xavier wants to retain you on the daughter's behalf on the hovering-while-intoxicated charge. Xavier would pay your legal fees and wants to set up a joint meeting among all three of you. Assuming Model R. Prof. Conduct 1.8 is unchanged from 2021, may you commence representation on this basis? A. Yes, as long as Xavier's daughter gives informed consent. B. Yes, as long as Xavier's daughter gives informed consent, you have opportunity to meet alone with Xavier's daughter to discuss what happened, and Xavier understands his daughter is the decision-maker. C. No, because a third party can never pay the legal costs of a lawyer's client. D. No, because parents may arrange for lawyers to be retained only for children who are minors.

B. Yes, as long as Xavier's daughter gives informed consent, you have opportunity to meet alone with Xavier's daughter to discuss what happened, and Xavier understands his daughter is the decision-maker.

Cain was a pedestrian injured in the car accident between Abel and Bain. Cain asks Stacia, a lawyer at the Furst & Furst law firm (Liang's prior law firm), to represent him in claims against both Abel and Bain. May Stacia do so consistent with the Model Rules of Professional Conduct and comments, class video, and office hour? A. Yes, as long as Stacia has no confidential information about the matter of Abel v. Bain. B. Yes, as long as no lawyer in the law firm has confidential information about the matter of Abel v. Bain. C. No, because an ethical screen was not put in place before Liang left Furst & Furst. D. No, because terms of the settlement agreement would prevent Stacia from obtaining informed consent, confirmed in writing.

B. Yes, as long as no lawyer in the law firm has confidential information about the matter of Abel v. Bain.

Seller retained Lawrence, an attorney licensed in North Dakota, to sell some farmland Seller owned in the western part of the state. Seller and Lawrence agreed in writing that the lawyer would be paid 3% of the sales price. Seller had owned the land for only two years. Seller provided Lawrence with paper copies of all the documents from her file regarding the land and her purchase of it in 2019. The documents included a 2018 property-tax statement indicating the property was worth $500,000 and a 2019 letter from a title-search company indicating there were no mortgages or other liens on the property. To a file folder containing the documents, Seller affixed a Post-it Note with the instruction: "Just get rid of the property for $400,000 or more and transfer what I'm owed into my bank account. I don't want anything to do with this land anymore." Lawrence advertised the farmland on a couple websites. With the advertisements, Lawrence provided pdf downloads of the 2018 property-tax statement and 2019 letter from the title-search company but did not comment on the documents. Upon seeing the advertisement, an oil speculator offered Lawrence $500,000 cash for the property. Lawrence called Seller to give her the great news. "Super work, Lawrence," Seller said. "Do whatever you need to do to get this done." Lawrence closed the deal, had Seller sign final sales documents, and transferred $485,000 into Seller's bank account (the $500,000 minus Lawrence's 3% fee). The next day, Seller flew to Costa Rica and cut off all ties with friends, family, and Lawrence. Unbeknownst to Lawrence and the oil speculator, Seller had taken out a mortgage for $450,000 and pocketed that amount. When six months passed without Seller making a mortgage payment, the mortgage company obtained possession of the land. The oil speculator cannot locate Seller, so he has sued Lawrence for legal malpractice. Based on this week's reading and office hour, does Lawrence risk liability? A. Yes, because the Model Rules of Professional Conduct prohibited Lawrence from advertising the property on a website. B. Yes, as long as the oil speculator can establish his reliance on Lawrence's advertisement and the pdf downloads was reasonable. C. No, because Lawrence acted diligently by posting the advetisement quickly. D. No, because the oil speculator was not Lawrence's client.

B. Yes, as long as the oil speculator can establish his reliance on Lawrence's advertisement and the pdf downloads was reasonable.

It is 2028 and you have been practicing criminal law for five years as a Stearns County Assistant County Attorney (prosecutor). On behalf of the government, you have resolved hundreds of cases through guilty pleas, including a matter in which a defendant, Yanni, was charged with drunken driving. Because it was a first-time offense, Yanni pled guilty and paid a fine in lieu of jail time. In return, Yanni agreed to typical probationary conditions: to not commit same or similar offenses for two years and to seek substance-abuse counseling. Two months after Yanni's case was resolved, you "switched sides" and joined a nine-lawyer law firm that does criminal defense and family law. A week into your new job, Yanni calls and asks you to defend her against federal money-laundering charges. Assuming the relevant Rules of Professional Conduct are controlling and are unchanged from 2021, may you do so? A. Yes, because the state and federal systems are completely separate and there can never be a conflict of interest. B. Yes, as long as you determine the federal and state matters do not involve the same basic facts and you do not reveal information relating to your representation of the state in its prosecution of Yanni. C. No, because it is per se unethical for a prosecutor to become a defense attorney. D. No, because the former client is the state, and it is impossible to get informed consent from a state.

B. Yes, as long as you determine the federal and state matters do not involve the same basic facts and you do not reveal information relating to your representation of the state in its prosecution of Yanni.

True or false: A lawyer-client relationship is formed only when the client gives informed consent, confirmed in writing.

False

Bill is a second-year associate at a small litigation firm in Minneapolis. Occasionally the firm is retained by New York-based Verizon, the cell-phone company, for smaller matters. Bill's assigning partner has assigned him to defend Verizon in a breach-of-contract matter. While the case is pending, Bill's cell-phone contract comes up for renewal. He is thinking about getting rid of T-Mobile and signing up with Verizon instead because of a "three months free" promotion. May Bill do so consistent with the Model Rules of Professional Conduct, comments, other assigned reading, and office hour? A. Yes, because by becoming a Verizon customer Bill would assure he does not have a concurrent conflict of interest. B. Yes, because Bill is not in a position to take advantage of Verizon. C. No, because Bill is entering into a business transaction with a client. D. No, because the "three months free" promotion has monetary value and therefore is a gift.

B. Yes, because Bill is not in a position to take advantage of Verizon.

Java and Joe were best friends from college who owned a coffee shop together. Each owned a 50% interest in the business. Unfortunately, they knew more about coffee than about business practices and failed to obtain commercial general liability insurance. Adding to the misfortune, a customer slipped on a wet floor mat and sued the friends and business partners for personal injuries. In the complaint, Java and Joe were co-defendants. A third college friend, Jill, agrees to represent them in the case, for $25 an hour. The friends are most appreciative. They tell Jill she can stop by for a free cup of coffee anytime, "every morning if you want." Jill obtains a settlement demand from the plaintiff, a surprisingly low figure. She calls Java with the good news. He says: "Sounds great, let's accept the demand. Joe asked me to deal with it. I'll let him know the good news and we'll just plan to each pay 50%." Jill calls back the plaintiff's attorney and accepts the settlement demand. Has Jill acted contrary to the Model Rules of Professional Conduct? A. Yes, because daily cups of coffee, in the aggregate, amount to a substantial gift. B. Yes, because Joe needs to know how much he might be paying. C. No, because Java said Joe provided informed consent. D. No, because under the doctrine of agency, Java has apparent authority to agree to the settlement on behalf of Joe.

B. Yes, because Joe needs to know how much he might be paying.

Miguel, a criminal-defense lawyer with 10 years of experience, represented Tom in a criminal case involving alleged embezzlement from Tom's employer. Miguel objectively believed there was a strong defense because of evidence indicating Tom's adult daughter stole the funds using a password she found on a Post-it Note on Tom's dresser. In other words, Tom lacked the requisite criminal intent. Tom was most appreciative for Miguel's assistance. Tom suggested they purchase a condo in Hawaii together, both to visit and to rent out to vacationers. Miguel stayed silent. However, the next day, Tom's real estate lawyer, Larry, called Miguel to propose a deal. When Miguel asked to see any relevant documentation, lawyer Larry sent Miguel a letter, on Larry's letterhead and signed by both the lawyer and Tom, proposing Tom and Miguel each pay 50% of the purchase price. The letter explained Larry represented Tom in the purchase. Applying the principles from the Model Rules of Professional Conduct, the comments, and office-hour discussion, may Miguel purchase the time share with Tom? A. Yes, because the transaction is unrelated to the criminal case so there can be no conflict of interest. B. Yes, because Tom's real estate lawyer contacted Miguel and Tom has signed off on the deal. C. No, because the criminal case is still pending and it would be unethical for a lawyer to do a business transaction with a potential criminal. D. No, because Miguel is not represented by separate counsel.

B. Yes, because Tom's real estate lawyer contacted Miguel and Tom has signed off on the deal.

Zeke is in his second year at Little Rock School of Law in Little Rock, Arkansas. Unfortunately, he skipped the assigned reading for the first week of Professional Responsibility class. The next week, Zeke conferred with the Financial Aid office to finalize paperwork for a fall loan application, a subject area he knew well. He already was thinking about how to pay off the loans after he passed the bar exam. Then he got a brainstorm: Why not start a website where he would answer questions about federal regulations that apply to law-school loans, and then offer to complete loan-application paperwork for a fee? Zeke then dropped out of law school, figuring he could make more with that business and his existing eBay business. If Zeke follows through on his plan, does he risk being punished under an Arkansas statute that forbids unauthorized practice of law? A. Yes, because the Legislature's statutory scheme regulating law-school loan applications has preempted the common law. B. Yes, because Zeke never became a lawyer. C. No, because the Arkansas Supreme Court has sole disciplinary authority over all matters related to law practice and could punish Zeke because he used to be in law school. D. No, because the loan applications implicate federal law and would require the court to apply the "reverse-Erie" doctrine.

B. Yes, because Zeke never became a lawyer.

During her third year of law school, Yvonne worked as a summer law clerk at Picture This PC, a law firm specializing in copyright infringement cases. The firm uses software to troll the web for unauthorized uses of photos. Once the software finds an infringing use, the law firm sends a "cease and desist letter" to the website's owner and threatens litigation unless payment is received. Yvonne did not do very well in Professional Responsibility. But she excelled in legal writing, so as a clerk she started writing the cease-and-desist letters, which her supervisor signed. At her supervisor's direction, Yvonne informed letter recipients that the "statutory minimum" for each infringing use of a copyrighted photo was $1,000, even though she learned in Copyright Law class the United States Code sets the minimum at $750 and leaves room for a court to assess as little as $200 per violation depending on the circumstances. Now Yvonne is licensed as an attorney and signs the letters. Her supervisor's directive remains in place. A letter recipient has reported her to the Board of Law Examiners for making false statements about the law. Is she subject to discipline? A. Yes, because making false statements about federal law is a crime and a lawyer's criminal acts always must be reported. B. Yes, because a subordinate attorney can be subject to discipline even when acting pursuant to a supervisor's direct instructions. C. No, because she acted at the direction of her supervisor, who is vicariously liable. D. No, because by identifying both $200 and $750 as the penalty, the federal statute is ambiguous, which in turn creates an arguable question about congressional intent.

B. Yes, because a subordinate attorney can be subject to discipline even when acting pursuant to a supervisor's direct instructions.

You are a third-year associate at a law firm that specializes in defense of civil cases and real-estate matters. Larry, Darrell, and Daryl come asking you to represent them jointly in a lawsuit over a bar fight. Each was sued for battery, and each is adamant that it is a case of mistaken identity. After making sure there are no conflicts of interest with respect to prospective, former, or current clients, you explain to each prospective client individually that there could be potential conflicts of interest if you represent all three of them. Larry, Darrell, and Daryl are adamant that their interests are aligned. Each says he was not even in the part of the bar where the fight occurred. Each of them signs a form consenting to the joint representation. Could you have an obligation to withdraw at some point? A. Yes, if a more-senior law partner starts representing the bar in a boundary line dispute. B. Yes, because the duty to withdraw can supersede the clients' written agreement if a conflict develops. C. No, because each client provided informed consent, confirmed in writing. D. No, because any crossclaim between or among Larry, Darrell, or Daryl would fail as a matter of law.

B. Yes, because the duty to withdraw can supersede the clients' written agreement if a conflict develops.

Corinne has represented Delilah and Delmar Richie in real estate matters. The Richies are in their late 60s. It is the second second marriage for Delilah, who came from a well-do-do family and inherited millions. It is a first marriage for Delmar, who was a middle-school social-studies teacher for 40 years before marrying Delilah.Delilah calls Corinne for help with the latest real-estate deal. Delilah wants to pay cash for a fancy lakefront property consisting of an 8,500-square-foot mansion and 2,000-square-foot guest house. The property was last assessed for $9.8 million is in foreclosure. Delilah's plan is for her 22-year-old great-nephew, Devin, to move out of his tiny apartment in St. Paul and live on the lake property where he will manage it as an Airbnb. This would give Devin some "spending money" while learning the "ways of business" and abandoning what is, in Corinne's mind, Devin's misguided goal to attend law school to become a public defender. All three -- Devin, Delilah, and Delmar -- would be "on the deed." Devin, Delilah, and Delmar come to Corinne's office and sign a standard retainer agreement agreeing to Corinne's $400-an-hour fee. Corinne has her assistant provide a copy of the signed agreement to each client. There are no other documents concerning the representation. Devin and Delmar head to the car.Alone with Corinne, Delilah whispers to the lawyer that the attorneys' fees invoices should go only to her and she is to be the "contact person" in the transaction. In subsequent emails, phone calls, and letters, Corinne communicates only with Delilah and sends monthly invoices only to Delilah.Has Corinne violated a Rule of Professional Conduct? A. Yes, because a 22-year-old lacks capacity to be involved in such a high-stakes purchase. B. Yes, because the retainer agreement did not limit the scope of representation regarding communications or who has decision-making authority. C. No, because Delilah is paying the legal fees and she and Delmar will pay cash for the property so Devin need not be informed. D. No, because Delilah made an oral offer about being the "contact person," which Corinne accepted by performance, so there was an enforceable oral contract.

B. Yes, because the retainer agreement did not limit the scope of representation regarding communications or who has decision-making authority.

A week after receiving his law license, Enrique opened a law practice targeted at cabin owners in the Brainerd Lakes Area of Minnesota. He named himself the "Minnesota Law Loon." On his website, he posted a video of himself running a speedboat into a dock, jumping out of the boat with an oar in his hand, smashing the oar against a boat window, and kicking the side of the boat while wearing flip-flops. All the while, Enrique is wearing a suit coat and tie, Bermuda shorts, and a hat fashioned after a loon, Minnesota's state bird. The website video ends with him making the following statement: "I grew up in the area and practically lived on the water. Nobody knows lakes like I do! Nobody is better-qualified to fight the birdbrains on the County Board more than I am when it comes to how you may use your lakefront property. If they think otherwise, they are certifiably crazy. I am the one certified to help you fight city hall. I'll smash them like a minnow. Go fishing? GO FIGHT!" Enrique is great at fishing, but he has never been to a zoning board meeting or represented a client. He never had an externship, internship, or clerkship during law school. But he is starting to get clients, and they are citing the outrageous albeit entertaining website for why they are hiring him. Does Enrique risk discipline associated with the website? A. Yes, because the references to "birdbrains" and smashed minnows are unprofessional when measured against Minnesota's conserative community standards. B. Yes, because there is a substantial likelihood prospective clients will believe he has experience and qualifications he does not have. C. No, because the website is a public service and the First Amendment gives Enrique license to speak freely by any electronic means. D. No, because any reasonable non-lawyer would recognize the satirical nature of the video and not reasonably rely on it for anything.

B. Yes, because there is a substantial likelihood prospective clients will believe he has experience and qualifications he does not have.

After you consult with your client George, you claim $1 million in damages on behalf of him. Then the matter of George v. Gus goes to trial. You explain the process of cross-examination and that sometimes you must question witnesses aggressively if their trial testimony differs from their prior deposition testimony. George appears to understand and says that although he is mad at his twin brother, "Gus is not a liar." However, on the witness stand Gus testifies falsely. You cross-examine him aggressively, which causes Gus to break down in tears in front of the jury and admit his lies. It is clear you made quite an impact and a jury verdict greatly exceeding $1 million is not out of the question. The judge halts the trial for a 10-minute recess. George asks to talk with you in the hallway. He is extremely irritated by the aggressiveness of your cross-examination and how you treated his twin brother. Then George says he "wants out of this mess" and directs you to demand $5,000 from Gus in exchange for George dismissing the case. You remind George about your prior discussion regarding cross-examination and try to reason with him about the status of the case. But then the 10 minutes are up and the trial is about to restart. The judge asks whether there are any "housekeeping issues," and you say no. The judge brings in the jury and Gus' lawyer calls the next witness. Based on the week's reading material and office hour with Professor Jeanetta, have you acted unethically? A. Yes, because you should have halted the trial to double-check that George was OK with the aggressive cross-examination. B. Yes, because trial has resumed without Gus having opportunity to know George would take $5,000 to settle the case. C. No, because no reasonable lawyer would settle a $1 million case for $5,000. D. No, because the ethical duty to discuss settlements applies only when a defendant makes an offer.

B. Yes, because trial has resumed without Gus having opportunity to know George would take $5,000 to settle the case.

Geraldine is general counsel for Burger Queen, a franchisor of fast-food restaurants. Burger Queen is headquartered in Florida and has regional offices around the country, including in Michigan. Geraldine is licensed to practice law only in Florida. During the pandemic, times have been tough for Rudy, a franchisee who operates a Burger Queen restaurant in Lansing, Michigan. Rudy leases the building from Burger Queen, which owns the Michigan property. However, Rudy hires employees, orders supplies, and generally operates the fast-food restaurant. Rudy can no longer afford to pay the rent. Burger Queen's district manager based in Michigan wants to negotiate payment terms with Rudy to avoid an eviction action in county court. The district manager, who is not a lawyer, has asked Geraldine for advice about what to say to Rudy. May Geraldine provide advice to the Michigan-based district manager, consistent with the Model Rules of Professional Conduct? A. Yes, because Burger Queen owns the building. B. Yes, but the best practice would be for Geraldine to associate with a lawyer licensed in Michigan to provide the advice, in case Rudy needs to be evicted. C. No, because Geraldine is not licensed in Michigan. D. No, because the negotiation is not a formal mediation or arbitration.

B. Yes, but the best practice would be for Geraldine to associate with a lawyer licensed in Michigan to provide the advice, in case Rudy needs to be evicted.

A week after Yanni called you she called another lawyer at the firm. The call came from the Stearns County Jail where Yanni was being held pending charges from a drunken driving arrest early that morning. May your colleague represent Yanni in the new drunken driving case? A. Yes, if Yanni gives informed consent, confirmed in writing, to the representation. B. Yes, if you are screened from participation in the matter, you receive no fee from the representation, and the Stearns County Attorney's Office is notified in writing. C. No, because all your prior work is imputed to the new law firm, which is disqualified as a matter of law. D. No, because Yanni failed to adhere to the conditions of probation, the Stearns County matter is revived and there is a non-consentable conflict.

B. Yes, if you are screened from participation in the matter, you receive no fee from the representation, and the Stearns County Attorney's Office is notified in writing.

Your client is QuikChange, an auto-repair business owned by a married couple. The co-owners are out of town and out of touch, touring Antartica. It was a well-earned vacation because they had spent a lot of time dealing with a competitor who came to town. The competitor is QuickerChange, which advertises five-minute oil changes "or your money back." The marketing slogan, coupled with the new business' deceptively similar name to your client, is making a dent in your client's business. The new arrival, QuickerChange, has been subject to numerous complaints on social media. As a result, the Attorney General Office's has commenced a civil enforcement action and is trying to shut down the business. Unfortunately, the Attorney General's Office has confused your client QuikChange with the new arrival QuickerChange. The Attorney General erroneously commences the action against your client. You are QuikChange's registered agent, meaning you are authorized to accept service of the Attorney General's summons and complaint. You page through the documents and have no doubt the Attorney General has confused the businesses. You see a Temporary Restraining Order (TRO) hearing will occur in an hour. You put on your "emergency" courtroom attire and arrive 20 minutes early, only to find the Attorney General himself holding a news conference outside the courtroom. Three TV stations are there, and two are live streaming the event. As the Attorney General makes false allegations against your client, you try but fail to reach the owners in Antartica. As the news conference ends and the Attorney General enters the courtroom, the journalists start to leave because cameras are not allowed in the courtroom. You shout and say this: "Everyone, stop! There is a great injustice being done here, and I need to make a statement." You then explain the Attorney General has sued the wrong business. Have you acted ethically according to the Model Rules of Professional Conduct, comments, and office-hour discussion? A. Yes, because the matter is not yet at the jury-trial stage. B. Yes, to counter the Attorney General's false claims in the minds of persons who would see the media report. C. No, because the constituents of QuikChange control the objectives of representation and they have not provided consent for the public statement. D. No, because you can make a judicial statement in court momentarily so there is no ethical basis for an extrajudicial statement.

B. Yes, to counter the Attorney General's false claims in the minds of persons who would see the media report.

After accepting the watch, litigation ensued and legal fees start piling up. Terry is $20,000 overdue on paying the bill. Still, the professional relationship between Sang and Terry remains amicable. During a Zoom meeting, Terry tells Sang he just watched an episode of "Antiques Roadshow," a television show that features appraisers assessing the value of people's hidden treasures. Terry says after watching the show he believes the watch is worth $50,000. Terry asks Sang to give it back to him. In response, Sang proposes that she loan Terry $20,000, hold the watch as "collateral," and apply the money toward Terry's outstanding balance until the watch can be appraised. The next day, Sang memorializes the proposal in a written document. Terry signs the document and sends back a pdf.Has Sang violated the Model Rules of Professional Conduct? A. Yes, because Sang failed to return a voidable gift. B. Yes, because Sang has provided financial assistance to a client. C. Both of the above, for the reasons articulated above. D. Neither of the above, because Terry gave written consent.

C. Both of the above, for the reasons articulated above.

Lawyers' common-law duties supplement and inform application of the Model Rules of Professional Conduct in many ways. This is particularly true in a conflicts-of-analysis. After a lawyer stops representing a client, which of these duties continue(s) in ways relevant to determining whether there are conflicts of interest? A. Duty of confidentiality. B. Duty of loyalty. C. Both of the above. D. Neither of the above.

C. Both of the above.

Tex represents Sloan, a Texas citizen who had $65,000 in belongings stolen from his room when he was staying at a Louisiana resort. The allegation is the resort was negligent. There is no diversity of citizenship and the amount in controversy does not exceed $75,000, so a civil action must be commenced in a Louisiana state court. In the written fee agreement, Sloan agrees Tex would take one-third of any recovery. Because Tex is not licensed to practice law in Louisiana, he sends an email to Louise, a law-school classmate licensed in Louisiana, asking her to draft and file the complaint in Louisiana and to serve the summons and complaint on the resort. Sloan is cc'd on the email and deletes it to keep his inbox clean. Five months later, when Sloan travels to Louisiana to be deposed, Sloan signs a written fee agreement stating that Louise would take one-third of any recovery. A month after that, the matter settles for $60,000. Sloan is surprised to receive a check for only $15,000. In a single email, he complaints to both Louise and Tex. In a series of "reply all" emails, Louise and Tex explain that Sloan's deposition cost $5,000 and he is responsible for paying that amount, and that Sloan agreed that each lawyer would take one-third of the recovery. Sloan sends several emails in protest, which the lawyers ignore. Instead, they each transfer $20,000 into their business accounts. According to the Model Rules, which among the following describes the ethical problem or problems? A. The fee is unreasonable. B. The lawyers did not put the $5,000 in a trust account. C. Both of the above. D. Neither of the above, because Sloan agreed to the arrangement.

C. Both of the above.

Frances represents Winifred in a pending divorce action from her husband, Hal. It has been terribly contentious, the worst Frances has ever seen. They have one 11-year-old daughter, Debra, who spends weekends with Winifred and the school week with Hal. From day one, it has been a challenge to manage Winifred's expectations because Winifred wants to litigate much of the case herself. Most recently, Winifred floated the idea of hiding a recording device in Debra's backpack on Sunday night before Hal picks up the girl for the school week. The device has a long battery life and significant memory and would capture a good share of the week's conversations. Winifred would reclaim the device from the backpack the following Friday when picking up Debra for the weekend. Based on legal research in a prior matter, you believe it is highly likely that hiding the recording device would violate a criminal wiretapping statute. What is your best course of conduct? A. Pretend you did not hear what Winifred said. B. Reiterate that by representing Winifred, you do not approve of her actions. C. Explain the potential ramifications of the act and try to persuade Winifred not to do it. D. Withdraw from the representation immediately and report Winifred to the police.

C. Explain the potential ramifications of the act and try to persuade Winifred not to do it.

Martha had practiced law in rural Upstate New York for nearly 40 years and was looking to ease out of the practice. She hired Justin, who grew up in Brooklyn and was newly admitted to the New York Bar. Martha was impressed by Justin's legal-research skills because she had not kept her legal-research skills up to date. Martha bought Justin a top-notch Lexis subscription with full access to all New York legal authority and told him to use it without limitation to investigate new practice areas. Martha's existing clients included farmers who were branching out into the increasingly lucrative vineyard business. Neither Martha nor Justin knew anything about the New York laws that apply to vineyards. But Martha liked her wine. During Martha's extended summer vacation to Minnesota, she tasted Minnesota-made wine at the Minnesota State Fair, loved it, bought 20 cases, stored it at her New York law firm, and headed to the Outer Banks of North Carolina for a month. Justin found and drank the Minnesota wine, liked it, and advised a law-firm client to include it on its tasting menu at the farm's New York vineyard. Justin did so without Martha's knowledge and without logging onto Lexis. He wanted his work to be a "surprise" for his new boss. Unbeknownst to Justin, New York has a statute prohibiting out-of-state wines at vineyard tastings. Because the client relied on Justin's advice, the client became subject to a New York State enforcement action. The client reported both Martha and Justin to the applicable New York Appellate Division, which disciplines New York attorneys. Based on the Model Rules of Professional Conduct, who is at risk of discipline and why? A. Martha, because she has direct supervisory authority over Justin. B. Martha, because she failed to keep her Lexis skills sharp C. Justin, because he gave legal advice without doing legal research. D. Both Martha and Justin, for the reasons above.

C. Justin, because he gave legal advice without doing legal research.

Everett loved constitutional law and wanted to do more work as a civil rights attorney, particularly on matters related to discrimination. He watched applicable YouTube videos that generally described the federal statutes that provide grounds for litigants to maintain constitutional cases in federal and state courts. Finally, he brought in his first client, someone who alleged she was fired because of racial discrimination. On a Friday, Everett told the client he would spend a weekend studying the specific law and not charge the client for that work. As Everett explained, although he had two years of law-practice experience, this was his first discrimination case. After a weekend of research, Everett came to believe a two-year statute of limitations applied to the client's case. He filed a charge with the U.S. Equal Employment Opportunity Commission within two years of the alleged violation. Unfortunately for Everett, a 180-day deadline applied and the EEOC rejected the filing as untimely. Everett discloses the error to his client, who sues him for legal malpractice. Thankfully, Everett had malpractice insurance. During the discovery phase in the malpractice action, it is discovered that Everett's former client was fired not because of discrimination, but because of embezzlement from the employer.Based on the week's reading and office-hour discussion, is Everett liable for legal malpractice? A. Yes, because Everett displayed lack of competence by failing to do basic legal research. B. Yes, because Everett failed to exhibit basic diligence as a matter of law. C. No, because Everett's former client was fired for non-discriminatory reasons. D. No, because Everett did not charge the client for the research that led to the error.

C. No, because Everett's former client was fired for non-discriminatory reasons.

Morgan and Nevada are best friends from college. Then they went their separate ways--Morgan to law school and law practice, and Nevada to become a golf pro. Both became successful in their careers. However, after a decade, both were looking for a change. They met at William's, a bar down the street from the law school. After more than a few drinks, they came up with a plan. Their plan was to form a for-profit corporation, buy an abandoned golf course, refurbish three holes, name it "Legal Eagles, Inc.," and offer both legal advice and golf-related advice--all for a flat fee of $200 per golfer per hole. Morgan, who would own 60% of the corporation, would offer the legal advice. Nevada, who would own 40% of the corporation, would offer the golf advice. Each golfer would sign an agreement that explicitly describes the arrangement. The written agreement also would explain the golfers "assume the risk of injury, including but not limited to golf balls and potential legal malpractice." According to the Model Rules of Professional Conduct, may Morgan offer legal advice under this arrangement? A. Yes, because Morgan would own a controlling interest in the corporation. B. Yes, because the golfers would give informed consent to the arrangement, confirmed in writing. C. No, because Nevada would own an interest in the corporation. D. No, because the rules forbid lawyers from forming corporations with friends as a matter of law.

C. No, because Nevada would own an interest in the corporation.

Tom is one of five lawyers in the legal department of CastCom, the local internet provider. He has received word that an employee has stolen smartphones from homes during internet installations and is continuing to do so. Tom begins interviewing employees to find out what they know. One employee, Iris, tells Tom she did not steal anything but she has a good idea who might be doing the thefts. She offers to tell Tom about the potential suspect if Tom agrees to keep Iris' name out of it. Based on the assigned reading and discussion during the office hour, may Tom make such a deal with Iris? A. Yes, because Iris is not the client so there is no conflict of interest. B. Yes, because the attorney-client privilege would protect Tom from having to disclose Iris' name to her manager. C. No, because Tom needs to be loyal to CastCom. D. No, because it is impossible to obtain informed consent, confirmed in writing, for an agreement to keep an identity private.

C. No, because Tom needs to be loyal to CastCom.

Sang, an attorney, visited the home of Terry, a prospective client and homeowner who had a property-tax dispute against the city. After getting preliminary information and calling her office to confirm there was no conflict of interest, Sang provided a retention letter to Terry that outlined the scope of representation, Sang's fee, and other material terms. While Terry was reviewing the letter, Sang casually commented on a pocket watch Terry had displayed on a fireplace mantel. Terry said he obtained it at a garage sale about 20 years ago for $5. Sang smiled and said: "It looks great where you have it displayed." Several months later, at holiday time, Terry had the pocket watch delivered to Sang's office along with a note of appreciation for Sang's hard work. Sang displays the watch in her office. Has Sang violated the Model Rules of Professional Conduct? A. Yes, because it is per se unethical for lawyers to accept gifts from clients. B. Yes, because by commenting on the watch while Terry was reading the engagement letter, Sang engaged in undue influence and solicited a gift. C. No, because based on what Terry told Sang, the gift was not substantial. D. No, because the note accompanying the watch consisted of Terry's informed consent for Sang to have the watch.

C. No, because based on what Terry told Sang, the gift was not substantial.

Abel was injured in a car accident. Abel sued Bain, who allegedly did not stop for a red light. Liang is a lawyer who represents Abel. She is at the law firm of Furst & Furst, a law partnership of five lawyers. An insurance company calls Liang, asking her to represent Bain in the litigation of Abel v. Bain. May Liang do so consistent with the Model Rules of Professional Conduct and comments, class video, and office hour? A. Yes, because the insurance company is the client so there is no conflict of interest. B. Yes, as long as Liang obtains informed consent confirmed in writing from Abel and Bain to submit the matter to binding arbitration. C. No, because it is unethical for law firms to represent both plaintiffs and defendants. D. No, because the conflict is non-consentable.

C. No, because it is unethical for law firms to represent both plaintiffs and defendants.

A client confidentially delivers a flash drive containing files of business records to his tax attorney to obtain the attorney's legal advice about taxes. The business records are routine bookkeeping files. After the IRS brings an enforcement action, the attorney asserts the documents are subject to the attorney-client privilege. Is the attorney correct? A. Yes, because the client delivered them to the lawyer to get legal advice. B. Yes, but only if the lawyer was reasonably anticipating litigation at the time. C. No, because the act of delivering the documents without more does not fall within the privilege. D. No, because there is no protective order.

C. No, because the act of delivering the documents without more does not fall within the privilege.

Lucinda is a licensed attorney who represents civil plaintiffs injured in car accidents and criminal defendants charged with drunken driving. Her solo-practice law firm is in a strip mall owned by a limited liability corporation ("LLC") over which she has complete control. The mall is next to the county courthouse. The mall's tenants include a certified public accountant, licensed chiropractor, and community-owned microbrewery. Yesterday was the first day the microbrewery's tap room was open after the pandemic. To celebrate, Lucinda treated the accountant and chiropractor each to a frothy pint, while social distancing of course. During their talk, Lucinda offered the chiropractor and accountant a $100 break in monthly rent on condition they each refer at least two clients a month to her. They both agree. Does the arrangement satisfy the Model Rules of Professional Conduct? A. Yes, because a pint of beer is comparable to a holiday gift. B. Yes, because Lucinda is doing business as an LLC. C. No, because the discount in the monthly rent has value. D. No, because Lucinda did not take steps to ensure the accountant and chiropractor provide only basic information about the prospective clients.

C. No, because the discount in the monthly rent has value.

You have been a law-firm partner with Avery for 20 years. There are eight other partners and ten associates. The past few years, Avery has focused her practice on defending dentists and other medical professionals against malpractice claims. Avery swings by your office to pose an ethical dilemma. She found out her own dentist has been sued for pulling a patient's healthy tooth instead of a decayed one. Avery finds it hard to believe her dentist could or would do such a thing. To make matters worse, one of your fellow partners in the office just agreed to represent the patient in the suit against Avery's dentist. Avery wants to defend the dentist pro bono. May she do so? A. Yes, because any conflict is based on Avery's personal interest. B. Yes, because the law firm can put up an "ethical screen." C. No, because there is a concurrent conflict of interest that is imputed to all lawyers at the firm. D. No, because the patient's case came into the law firm first and law firms follow the "first filed" rule.

C. No, because there is a concurrent conflict of interest that is imputed to all lawyers at the firm.

Mario, a new attorney licensed in New York, thought his dream job was to make a salary approaching $200,000 a year at a New York City litigation firm. But during the 2020 coronavirus pandemic, his duties were limited to 10 hours of document review each day, from the cramped confines of his efficiency apartment. As a result, he started watching a lot of Netflix, binged on a short series featuring a public defender, and decided to make a career switch to criminal defense. Three years later, after the pandemic, Mario had a criminal trial in federal court in which his client elected to testify, despite Mario's advice to not testify. Five minutes before Mario's client was to take the witness stand, the client leaned toward Mario and whispered in his ear he was going to admit to the crime in front of the jury. Mario was blindsided and disappointed because he knew there was a good chance of an acquittal if the client would only stick to the story. When the judge entered the courtroom and asked whether counsel had issues to discuss before the jury was brought in, Mario rose and said: "Your honor, respectfully, I am withdrawing from my representation at this time, effective immediately." Based on the Rules of Professional Conduct, the week's reading, and our office-hour discussion, it is likely the judge will grant the motion? A. Yes, because the representation is "at will" and the lawyer may terminate representation at any time. B. Yes, because Mario now has actual knowledge that his client persisted in committing a crime. C. No, because there is not good cause for withdrawal. D. No, because an unpaid fee is not the reason for the withdrawal.

C. No, because there is not good cause for withdrawal.

Parker and Riley, ages 21 and 23 respectively, grew up in Smalltown, New York. They came to know Orion, one of the town's few lawyers, who practiced law in a wide range of areas. Unfortunately, Parker and Riley were involved in a car accident. Parker was driving and looking down at his phone to type a one-word answer to a text message when another car swerved. Parker was forced to take the ditch. The other driver stopped and was legally intoxicated. It is not clear whether the drunken driver had insurance. Despite being cited for illegal cell phone use, Parker's injuries were relatively minor. However, Riley was more seriously injured. They both had auto insurance under their parents' insurance policy. Orion knows it is an attorney's fiduciary duty to maximize recovery for clients, including from insurance proceeds. Parker and Riley trust Orion as a lawyer. They are adamant the attorney represent both of them in a single civil action against the intoxicated driver. Besides claims against the driver based on negligence and negligence per se, Parker wants to maintain a claim for negligent infliction of emotional distress to recover damages for seeing Riley be seriously injured. May Orion represent the siblings consistent with the Model Rules and the week's office hour? A. Yes, because Parker and Riley are related by blood so there is no conflict of interest. B. Yes, as long as Orion obtains a joint confidentiality agreement in which Parker and Riley agree confidential information may be shared. C. No, because to maximize the monetary recovery Riley probably will have to sue Parker for negligence. D. No, because it is illegal to have siblings on a single insurance policy

C. No, because to maximize the monetary recovery Riley probably will have to sue Parker for negligence.

It is 2027 and you have been a licensed attorney for five years. You recently purchased a new home and are eager to meet neighbors at National Night Out. You strike up a conversation with Eric, who lives three houses away. You move to the side and have a one-on-one conversation. Eric asks what you do for a living. You explain you are a lawyer who works at the corporate office of a large consumer-products retailer. Eric says he shops there. You say: "It's a nice place to work." Next, Eric confides in you that his spouse, Robin, has been -- in his words -- "abusive in ways I would rather not discuss." You take that as a sign to move the conversation to other matters, and you do. Eric says nothing else about the matter. You tell no one about what Eric said. Three weeks later, you are walking home from the bus stop. You notice Eric and Robin are having a party. They are inside and you see no one. You hear yelling and the sound of glass breaking. You get to your front porch and dial 911. When the emergency dispatcher asks why you are calling, you say you hope you are wrong, but you think there could be a domestic incident occurring at Eric and Robin's home and want to call to be "on the safe side." When the dispatcher asks for the basis of your knowledge, you tell about the sounds you heard. When the dispatcher presses you for more details, you reluctantly volunteer you have information about potential "abuse" at the home. Based on the assigned rules, comments, and readings, have you committed an ethical violation? A. Yes, because you disclosed confidential information belonging to a prospective client. B. Yes, because the 911 call was based on circumstantial evidence and you risked making a false police report. C. No, because you did not discuss with Eric the possibility of forming an attorney-client relationship. D. No, because a reasonable person knows a corporate lawyer does not take on private clients.

C. No, because you did not discuss with Eric the possibility of forming an attorney-client relationship.

You are in a law firm with one other partner. You represent Sue in a contingency-fee matter and have done preliminary investigation on whether to recommend starting a lawsuit. As part of the investigation, you have consulted with an expert witness from whom you are awaiting a final written report and invoice, to be paid by the client. Then you and your law partner have a change of plans. Because of numerous insurance-related issues stemming from the 2020 pandemic, you and your partner decide to shift your law practice entirely to cases involving interpretation of insurance policies that provide "business interruption" insurance. As a result, you will not handle contingency-fee matters anymore. Based on the week's readings and office hour, which is a step you should NOT take during the process of withdrawing from the representation of Sue? A. Drafting a pro se complaint. B. Informing Sue of the specific date when the statute of limitations would bar a claim. C. Requiring Sue to pay the expert's bill before providing Sue with the client file. D. Providing Sue with referral to other counsel, preferably the names of around three attorneys.

C. Requiring Sue to pay the expert's bill before providing Sue with the client file.

You and Selena worked together at a large Chicago law firm. You both were among newer attorneys who worked on a complicated but fulfilling pro bono matter seeking legal remedies for children detained away from their immigrant parents. The matter was filed in a federal court in Minnesota. Neither you nor Selena was licensed in Minnesota. Nor were you admitted to practice in Minnesota's federal court. But your supervisor, Boris, had the necessary license and admission and you both worked on the case under his supervision. You and Selena left the law firm for other work -- you to work as a law clerk in federal court in South Dakota, Selena to do marketing for LexLaw, a leading provider of legal-research tools. During a lunch break at your new job, you get this voicemail message from Selena: "Hey, it's Selena. I'm at our old law firm doing a marketing visit. I was surprised to hear Boris has left the firm. No word on where he went. Anyway, our former assistant, Adam, gave me an envelope marked 'personal and confidential.' It was addressed to Boris but had a return address from one of the clients in the pro bono case we worked on. Adam said he doesn't know what happened to Boris, but as far as Adam knows the pro bono case is still ongoing. I'll be at the firm until 3:00. Could you call me back and let me know what you think I should do?" When you return Selena's call, and considering the week's reading and office-hour discussion, what should you tell her and why? A. Selena should tell Adam to try to find Boris, but if Boris cannot be located the letter should be returned to sender because it is marked "personal and confidential." B. Adam should find Boris' former legal assistant, and give the letter to that person to deal with because Boris has an ongoing ethical obligation to supervise his former legal assistant. C. Selena should take the letter to the law firm's managing partner, explain the situation, and recommend they open it together because there is an ongoing fiduciary duty to the pro bono client. D. Selena should do nothing because she is not practicing law and neither are you.

C. Selena should take the letter to the law firm's managing partner, explain the situation, and recommend they open it together because there is an ongoing fiduciary duty to the pro bono client.

After becoming a licensed attorney, Sona wasted little time setting up a website to begin practicing law. On the website, she offered services specifically to small-business owners who struggled financially during the pandemic. The website included this paragraph: Has your business been denied compensation from insurance companies? Are the insurance companies wrongfully claiming your business' insurance policies do not apply to pandemic-related losses? I am a lawyer who can help, and I have reasonable rates. Below the paragraph was an icon with the words "click here for a free consultation." When visitors to the website clicked on the icon, they were taken to a live Zoom meeting where Sona obtained as much information as she could. Attorney-licensing authorities in her state began an investigation. From among the following, what is Sona's best defense that what she did satisfied the Model Rules of Professional Conduct? A. Sona is a brand-new attorney so by defintion she has no conflicts of interest. B. The First Amendment applies to websites, which the attorney-licensing authorities may regulate under only when there is a compelling reason to do so. C. Sona's website was targeted at owners of businesses that hired lawyers during the process of corporate incorporation. D. Because of social-distancing requirements, Sona's use of Zoom was reasonable.

C. Sona's website was targeted at owners of businesses that hired lawyers during the process of corporate incorporation.

You are a second-year associate at a small litigation firm in Minneapolis. There are two partners and two other associates. Occasionally the firm is retained by California-based Pear, Inc., a computer manufacturer, for smaller matters. Your main contact at Pear, Inc. is Newton, who worked at Pear for 10 years as a paralegal and just received his law license last month. Newton has provided you with a stack of documents to be produced (provided) in discovery during the litigation, with a note indicating he has not had time to review them himself. In reviewing the documents, you see indications persons at Pear, Inc. might have been using illegal debt-collection techniques. Based on the Model Rules, comments, and discussion during the office hour, what is your best course of conduct? A. Immediately contact the chairperson of the Pear, Inc. Board of Directors, to satisfy your duty to "report up" and guard against assisting a client with a crime. B. Report Newton to the attorney licensing authority in his state to prevent any unethical conduct from being imputed to you. C. Talk to a partner at your law firm to discuss what you found and what to recommend to the client as next steps. D. Scan pdfs of the documents and save the files in a safe place, in case you ever have to "report out."

C. Talk to a partner at your law firm to discuss what you found and what to recommend to the client as next steps.

An attorney is helping a beer manufacturer prepare for trial involving a truck accident resulting in many deaths. To prepare the defense, the attorney sends a single email to give initial thoughts on potential defense strategies. The recipients of the email are the company's CEO and an accident-reconstruction expert the attorney hired on behalf of the client. During the litigation, the Plaintiffs ask for a copy of the email. The defense attorney cites the attorney-client privilege and work-product doctrine as reasons why the email is not disclosed. The plaintiffs' attorneys assert that because the email was sent to someone other than a client, the privilege and doctrine do not apply. The plaintiffs' attorneys ask a court to compel production of the email. How should the court rule and why? A. The court should order the email to be disclosed because it was sent to a non-party. B. The court should order disclosure because the email was sent to a non-lawyer. C. The court should deny production on grounds that the work-product doctrine protects it. D. The court should deny production because the reconstruction expert was acting in the capacity of an attorney.

C. The court should deny production on grounds that the work-product doctrine protects it.

During this week's office hour, we discussed the rationales for why there are ethical rules that govern lawyers' contacts with represented and unrepresented parties. The rationales are not based solely in what you are learning in Professional Responsibility. Which other law school classes help you appreciate and understand the rationales? A. Evidence, to learn about statements against interest. B. Business Organizations, to learn about corporate structure and decision-making. C. Constitutional Criminal Procedure, to learn about the rights of criminal defendants. D. All of the above.

D. All of the above.

In a criminal matter, which of the following are decisions the client and not the lawyer must make? A. Whether to plead not guilty. B. Whether to waive a jury trial. C. Whether to testify at trial. D. All of the above.

D. All of the above.

When must an attorney withdraw from a client's representation according to the Model Rules of Professional Conduct? A. When the representation violates a Rule of Professional Conduct. B. When the lawyer has a condition that materially impairs the lawyer's ability to represent the client. C. When the client discharges the lawyer. D. All of the above.

D. All of the above.

You are in your fourth day as a new associate at law firm with 100 lawyers. Just what you vowed would never happen has happened, less than a week into the job. You have sent an email to another lawyer containing the wrong attachment. The attachment is an office memo that contains your supervising attorney's redlined changes and comments about areas you should explore more deeply. The comments carry notations identifying your supervising attorney as the author of the comments. To make matters worse, you sent the email and attachment to opposing counsel -- a lawyer on the "other side" in that specific matter. Now they know your supervisor's mental impressions. To make matters even worse, you are working remotely. Based on the Model Rules of Professional Conduct and discussion during the office hour with Professor Lacy, what should you do? A. Nothing, and hope for the best while preparing for the worst (e.g., getting fired). B. Email opposing counsel immediately to request destruction of the document, making sure to "cc" your client and your supervisor. C. Obtain the client's informed consent for disclosure of the privileged information. D. Answer Call your supervisor, and if she is not available send an email asking to talk as soon as feasible.

D. Answer Call your supervisor, and if she is not available send an email asking to talk as soon as feasible.

Sue represents a motorist injured in an accident at Summit Avenue and Victoria Street in St. Paul. Sue serves a summons and complaint on Durante, alleging he ran a red light and caused the collision. Durante sees Sue's name and telephone number on the complaint and calls her in a panic. After identifying himself, Durante says: "How much money is this going to cost me?" Which of the following responses is most consistent with the Model Rules of Professional Conduct? A. Provide Durante with a settlement range for this sort of case, but aim on the high side in furtherance of Sue's fiduciary duty to her client. B. Transfer the call to her assistant to avoid becoming a testifying witness. C. Tell Durante the answer depends on whether he has insurance. D. Ask whether Durante has a lawyer.

D. Ask whether Durante has a lawyer.

You have practiced litigation for four years and have been meaning to branch out into construction law, specifically by representing homeowners who purchased new homes that suffer from leaking roofs and windows, wet basements, and other problems. You have brought in your first client, a married couple who raised kids, sold their large home, and bought a new condominium to downsize. Unfortunately, the windows leak. As you negotiate the engagement letter with your new clients, they say they want their son, Weslix, who works as a lawyer for the State Department of Transportation, to do the applicable legal research to save money. They point out Weslix has eight years of legal experience, all drafting and negotiating contracts for the the state agency. You check Weslix's LinkedIn profile and there is no indication he has litigation experience. What do you do? A. Send Weslix your Westlaw password. B. Call Weslix to discuss the arrangement, because the clients control the means by which the objectives of the representation are pursued. C. Ignore the request. D. Discuss with your clients the reason for legal research and why it will be important for you to do it in this case.

D. Discuss with your clients the reason for legal research and why it will be important for you to do it in this case.

A lawyer plays on a church softball team. After games, she routinely goes out for pancakes with fellow team members, asks about their legal woes, and provides her thoughts over second cups of coffee. In return, her fellow team members chip in to pay the tab. The lawyer is most likely: A. Providing voluntary pro bono publico service because the church is a nonprofit organization and the nonprofit status is imputed to the members. B. Not providing voluntary pro bono publico service because there has been no written representation agreement. C. Not practicing law, because "providing thoughts" over coffee does not consist of giving legal advice. D. Giving legal advice to accidental clients.

D. Giving legal advice to accidental clients.

You practice law at an eight-lawyer law firm that specializes in family law and trust-and-estates work. Two of the lawyers have a contract with the county to represent the best interests of minor children when the county seeks to put children into foster care. You focus your practice on estate planning. In response to your law-firm website biography, a married couple send you an email seeking your legal advice on estate planning. In the email, they explain it is the second marriage for one spouse who has a child from the prior marriage, and it is first marriage for the other spouse who has no children. They want to come to your office, and you agree. You tell them to bring all the documents they feel are relevant. When they arrive, what is the best course of conduct based on what you have learned thus far in the semester? A. Invite them into your office, page through the documents they brought, explain the ramifications of a joint representation, and ask them to sign a document providing informed consent to the joint representation. B. Invite them into your office, ask basic information about the child, and proceed only upon determining no one in your law firm has represented the child. C. Have them wait in the lobby while you page through the documents to determine whether there is a conflict of interest. D. Meet with one spouse individually, discuss the ramifications of a joint representation, and recommend a joint confidentiality clause. Then repeat the process for the second spouse.

D. Meet with one spouse individually, discuss the ramifications of a joint representation, and recommend a joint confidentiality clause. Then repeat the process for the second spouse.

According to the Model Rules of Professional Conduct, a contingent-fee arrangement is permitted for which of the following type or types of representation? A. Defense against criminal charges, as long as the fee agreement is in writing. B. A claim for spousal support, as long as the fee is reasonable. C. Both of the above. D. Neither of the above.

D. Neither of the above.

Ben grew up in Moorhead, Minnesota, a community on the east side of the Red River of the North. Across the river to the west is Fargo, North Dakota. Ben went to law school in North Dakota, received a North Dakota law license, and worked for the state government in Bismarck, the state capital, for three years. Now he wants to buy a house in Moorhead and open a solo law practice. He is licensed only in North Dakota. He intends to rent office space in downtown Fargo. He also intends to enter into an office-sharing arrangement in downtown Moorhead where he would rent a conference room by the hour to meet with clients. On his law-firm website, he intends to advertise he is "able and willing to represent clients from across the region." The website will identify both his Fargo office and Moorhead conference room as places where he can meet clients. For Minnesota cases, Ben plans to refer them to a lawyer licensed in Minnesota in exchange for referral fees. Can Ben rest assured he is satisfying the Model Rules of Professional Conduct? A. Yes, because the rules allow sharing fees with fellow lawyers. B. Yes, because renting a Minnesota conference room by the hour is not a systematic and continuous presence in Minnesota. C. No, because solo attorneys are forbidden from having two offices. D. No, because Ben risks representing to the public he is licensed and admitted to practice law in Minnesota.

D. No, because Ben risks representing to the public he is licensed and admitted to practice law in Minnesota.

Tia, an attorney, represented Cal, who was interested in purchasing a nicely remodeled lake residence as a second home. Without Cal's consent, Tia typed the property's address into Zillow (a free website that provides information on the market value of real property). When she did so, Tia found a home three doors away that was marketed as a "fixer-upper." The residence needed a lot of work but was in a great location, an even-better location than the home Cal wanted to buy. She read the property description to learn that the county would require the buyer to make repairs to bring the residence "up to code." At that moment, Tia became interested in purchasing and refurbishing the fixer-upper. Tia says nothing about the fixer-upper to Cal. Tia represents Cal in the purchase of the remodeled home, records the deed, and sends Cal a disengagement letter. A week later, Tia hires her own lawyer to represent her in the purchase of the fixer-upper that is three parcels away from Cal's home. With her lawyer's help, Tia completes the purchase. With respect to Tia's representation of Cal, has Tia acted unethically under the Model Rules of Professional Conduct and comments? A. Yes, because Cal did not provide informed consent in a signed writing. B. Yes, because Tia's efforts in bringing the residence "up to code" are reasonably likely to drive up the area's property values, to Cal's detriment. C. No, because Tia obtained separate counsel for the transaction. D. No, because Tia did not use the Zillow information to the disadvantage of her client.

D. No, because Tia did not use the Zillow information to the disadvantage of her client.

Tiberius gave it his all, but he lost at trial on behalf of a client in a difficult case of commercial litigation. He had managed client expectations well by telling his client the case was a long shot, and the client understood and was appreciative for the work. When paying Tibuerius' outstanding legal fees, the client attached a Post-it Note to the check reading: "Thanks again. Let me know if there's anything more I should know on my end." Two weeks later, the court entered final judgment. Because the client had paid the last bill in full, Tiberius forwarded a pdf of the Notice of Entry of Judgment with an email saying: "I hope things are well. See attached." Sixty-five days later, the client was having lunch with a friend from college, who happened to be a lawyer, and asked about whether the client had appealed. The client did not know what the friend was talking about, so called Tiberius for guidance. Tiberius was overcome with a sinking feeling upon realizing an appeal must be filed within 60 days of entry of judgment. The deadline had passed five days earlier. Has Tiberius provided representation consistent with the Model Rules of Professional Conduct? A. Yes, because the matter ended when the client paid the final bill and Tiberius' forwarding of the Notice of Entry of Judgment was pro bono. B. Yes, because he managed client expectations by telling the client the case was a long shot. C. No, because a lawyer is required to send a Notice of Entry of Judgment by certified mail, with an affidavit of service. D. No, because Tiberius had a fiduciary duty to discuss a potential appeal, particularly because the client sought the lawyer's advice about whether there's "anything. more I should know."

D. No, because Tiberius had a fiduciary duty to discuss a potential appeal, particularly because the client sought the lawyer's advice about whether there's "anything. more I should know."

True or false: A person who communicates with a lawyer for the purpose of disqualifying the lawyer is a prospective client.

False

During law school, Deshi took Criminal Law, Constitutional Criminal Procedure, and Evidence but took no advanced criminal law classes and had no internships or externships in the area. Then Deshi took the bar exam. While awaiting the results, Deshi was offered a position as an Assistant County Attorney in a small county about 200 miles from the closest metro area. About 75% of the position would involve criminal prosecution, including jury trials. Deshi's would-be supervisor, the elected County Attorney, spends most of her time advising the County Board and on other transactional matters. The County Attorney has appeared in court only once in five years. A former criminal-defense attorney is handling the county's criminal prosecutions but wants to retire completely from law practice within 18 months. Deshi's job is contingent on her passing the bar exam. It is Deshi's only concrete employment offer so far. She would not start work until and unless she passes the bar exam. Does Deshi risk disciplinary action if she takes the job, passes the bar exam, and starts practicing? A. Yes, because lawyers owe fiduciary duties to their clients to practice only in areas they studied in law school. B. Yes, because it is unethical to accept employment before being sworn in as a licensed attorney. C. No, because the Rules of Professional Conduct apply only to civil law practice, not criminal law practice. D. No, because in law school Deshi learned how to spot issues and evaluate evidence, and she will be able to consult with a lawyer experienced in criminal matters.

D. No, because in law school Deshi learned how to spot issues and evaluate evidence, and she will be able to consult with a lawyer experienced in criminal matters.

The brakes of a beer truck fail and it collides with a bus of senior citizens heading to a museum, causing multiple deaths. A lawyer visits the truck driver in the hospital, explains he has been hired to represent both the truck driver and the driver's employer at no cost to the driver, and asks the driver to write down everything she remembers so the lawyer can do legal research. The truck driver complies and shows no one else what she has written. The lawyer takes the writing to the law office and hands the handwritten note to her assistant so it can be placed in the client's file and also scanned into the law firm's electronic database. The note is not altered in any way. When lawsuits begin, a plaintiff's attorney asks the truck driver's attorney to disclose a copy of what the truck driver wrote so the plaintiff's attorney can ask questions about the document during a deposition and at trial if necessary. The defense attorney refuses to hand over the document. The plaintiff's attorney files a motion to compel the defense attorney to provide a copy of the writing. Will the court order disclosure? A. Yes, because the attorney-client privilege protects only writings by lawyers, not by clients. B. Yes, because litigation was not commenced until after the truck driver wrote the statement. C. No, because the document became attorney work product as soon as it was scanned into the law firm's computer system. D. No, because it is a privileged communication.

D. No, because it is a privileged communication.

You are the legal-ethics expert at a medium-sized law firm that represents municipalities such as cities and counties. The firm gives advice to municipalities and also litigates claims on behalf of them. Among the clients is Springfield County. Last month, a third-year associate at the firm commenced an action against the former Springfield County Administrator to recover money the administrator stole from the county. It is the third-year associate's first "big" case, and he needs your help. He has just been served with a Motion to Disqualify. It comes from the lawyer representing the County Administrator, who takes the position the entire law firm has a conflict of interest. Specifically, the lawyer alleges a senior partner at the law firm had extensive conversations with the former County Administrator about a controversial bridge project and gave legal advice about the project. Therefore, the entire law firm is "conflicted out." Based on the information thus far, assigned reading, and office hour, is there a valid basis for the Motion to Disqualify? A. Yes, because there is a personal conflict of interest. B. Yes, because the law firm has significantly harmful information about the former County Administrator. C. No, because the rules of imputation do not apply when the government is a party. D. No, because the County Administrator was not a client.

D. No, because the County Administrator was not a client.

You are licensed to practice law in Minnesota. For the first time, you are appearing solo (without a supervising attorney) at a mediation. You are nervous for several reasons, including because your client is with you and knows this is your first time doing a mediation alone. To make matters worse, your client went to law school but never practiced--but yet pretends she knows every last thing about litigation and mediation. The mediation session is in Duluth, Minnesota, just a couple miles from the Wisconsin border. It is a pre-suit mediation where the parties are trying to resolve the matter before a lawsuit is filed. The opposing party is a family business from Superior, Wisconsin. The business usually is represented by a Wisconsin attorney but for purposes of this matter only retained a Minnesota attorney. On the morning of the mediation, the Minnesota attorney was called away on a family emergency. Only the Wisconsin attorney arrived. Your client wants you to object to the Wisconsin attorney's involvement, on grounds the Wisconsin attorney is practicing law in Minnesota without a license. Under the Model Rules, should you follow your client's directive and object? A. Yes, because the Wisconsin attorney is not licensed in Minnesota and therefore is violating the Model Rules. B. Yes, because the client always controls the means of the representation. C. No, because having a non-litigator on the other side of the negotiating table benefits your client. D. No, because the Wisconsin lawyer's services are reasonably related to the Minnesota mediation.

D. No, because the Wisconsin lawyer's services are reasonably related to the Minnesota mediation.

Harriett went to law school with the goal of advocating for employees who face discrimination at work. Upon graduating, those jobs were hard to get. So Harriett reluctantly accepted a position in a sizable corporation's Human Resources Department. Harriett's goal was to learn the ropes of employment law while taking advantage of the corporation's generous 401(k) match. In a few years, she hoped to quit that job, join a plaintiffs' firm, and fulfill her dream of representing plaintiffs. One day, Harriett was using a back entrance to the corporate cafeteria. She encountered a cafeteria worker who was injured after slipping and falling in the kitchen. Although the cafeteria worker was not gravely injured, he was upset. Harriett then said, "Look, I'm a lawyer from HR. You can trust me with the truth." The cafeteria worker said he slipped on water. When Harriett pressed the worker for specifics, he acknowledged he and another employee were having a "water fight" with two dish-washing hoses at the time. Harriett responded, "Well, kitchens are stressful places and you needed to blow off steam, I'm sure." Then Harriett headed straight to the Corporate Safety Director's office to tell him the employee was injured in a slip-and-fall and to seek compensation for the injured worker. In doing so, Harriett omitted information about the water fight. Based on the assigned rules, comments, and readings, has Harriett acted ethically? A. Yes, because Harriett had a moral duty to make sure injured people are fairly compensated. B. Yes, because Harriett gave legal advice during an emergency. C. No, because in that moment the cafeteria worker had diminished capacity. D. No, because the cafeteria worker reasonably could assume Harriett was his lawyer.

D. No, because the cafeteria worker reasonably could assume Harriett was his lawyer.

Instead of calling Liang, the insurance company calls Pedro, a partner at Furst & Furst and a friend of Liang's from law school, asking Pedro to represent Bain in the litigation of Abel v. Bain. May Pedro do so consistent with the Model Rules of Professional Conduct and comments, class video, and office hour? A. Yes, because the insurance company is the client so there is no conflict of interest. B. Yes, as long as Liang is screened from the file and fee. C. No, because Pedro is friends with Liang and has a personal conflict of interest. D. No, because the conflict is non-consentable.

D. No, because the conflict is non-consentable.

Ben Sparrow and Beth Cardinal come to you seeking joint representation to file a Family Court petition to end their marriage after 12 years. They remain amicable and in fact friends, but have grown apart as each focused on career more than the relationship. They have been maintaining separate rented apartments in separate cities, have no children, own their cars and other property, and maintain separate bank accounts and retirement funds. They just choose to move on and not pay a bundle in legal fees when doing so. You sense a conflict of interest but Sparrow and Robin are adamant. You tell them to come back tomorrow. In the meantime, you draft a joint representation agreement in which they would agree to a joint confidentiality agreement, agree to give informed consent to waiving the conflict of interest, and also waive any right to assert claims against you based on the conflict. You also prepare a draft of the petition, captioned Sparrow v. Cardinal. The couple come back the next day and enthusiastically sign the agreement. May you represent them? A. Yes, because there are no child-custody or property-division disputes. B. Yes, because Ben and Beth provided informed consent in writing. C. No, because Ben and Beth would pay the fees jointly, and the Rules of Professional Conduct prohibit a lawyer from accepting shared fees from one or more clients. D. No, because the representation will require Ben ad Beth to assert claims against each other.

D. No, because the representation will require Ben ad Beth to assert claims against each other.

Polly is the County Attorney in Gravel County, Minnesota, population 3,500 residents. The County Attorney job is only part time and Polly runs the office out of her own basement. Under state law, criminal prosecutions and civil matters are among the county attorney's duties. Ron, a semi-retired lawyer who moved from the East Coast, works 10 hours a week out of Polly's basement. He focuses on representing the county in minor civil matters. Polly handles the criminal prosecutions. Recently, Gravel County experienced its first homicide in 15 years. To make matters worse, the victim was Ron's twin brother, Ralph, who was known around town for making a mint on Wall Street and then moving to rural Minnesota. The Gravel County Sheriff had neither the resources nor experience to investigate a homicide, so the Minnesota Bureau of Criminal Apprehension took over in conjunction with the Minnesota Attorney General's Office. Ron took possession of his deceased brother's belongings. They included Ralph's handwritten journal, which Ron left on his small desk in Polly's home office. One evening, Polly got nosy and started reading the journal. There was an entry in which Ralph wrote he had "gotten away" with a financial crime in New York fifteen years ago. Tucked inside the journal was a newspaper clipping detailing another person's arrest for the incident. Polly checked an FBI database to learn there had been a prosecution in the United States District Court for the Southern District of New York. Someone other than Ralph confessed to the crime, pled guilty, and served four years in prison. Polly checked a database of federal inmates to learn the person was released years ago. Google leads her to the person's blog on which he maintains his confession was coerced and he was innocent. Polly breathes a sigh of relief, thinking "what's done is done." The next day she attends Ralph's funeral to honor his memory and support Ron. She never tells a soul about what she read and learned. Has Polly acted ethically consistent with the Model Rules of Professional Conduct, comments, and office-hour discussion? A. Yes, because Polly owes no professional duty to persons in New York. B. Yes, because Ralph is dead and cannot be charged with a crime. C. No, because by reading the journal without obtaining a warrant, the journal entry is "fruit of the poisonous tree" under the Fourth Amendment and would be excluded from a federal prosecution. D. No, because there is a reasonable likelihood the wrong person was convicted of the offense.

D. No, because there is a reasonable likelihood the wrong person was convicted of the offense.

It is October 2023 and you have just been sworn into the bar to become a licensed attorney. Life is great because you have three job offers and will decide which one to accept on Monday. On your way home from the swearing-in ceremony, your college acquaintance Beau sends a congratulatory text message. You see the message upon getting home, and although you have not heard from Beau in six years, you reply: "Thanks Beau! What a surprise to hear from you. But yeah, I can't wait to start practicing! I hope to start my law career in two weeks!" Beau responds: "Great news and perfect timing! I need your help." Beau then proceeds to write three paragraphs about a landlord dispute and his potential eviction. Beau texts you a picture of a summons requiring him to appear in court in two days and finishes the text with: "So u will help, right? I'll pay u. Just let me know the $." You learned in Professional Responsibility class that it is highly unwise to practice law without malpractice insurance, even for a licensed attorney, which you have been for nearly an hour. You have no malpractice insurance, and for that reason (among others) you choose to ignore Beau's text completely and do not respond. Besides, you hardly knew him. Truth be told, you found him to be rather annoying. Assuming the 2023 Model Rules of Professional Conduct are the same as in 2021, do you risk discipline? A. Yes, because you did not send a nonengagement letter. B. Yes, because Beau asked a question about representation and it was your fiduciary duty as a licensed attorney to provide a response. C. No, because you had not yet been hired by a law firm or other legal employer. D. No, because there was no consultation between you and Beau within the meaning of the Rules of Professional Conduct.

D. No, because there was no consultation between you and Beau within the meaning of the Rules of Professional Conduct.

Nile Delivers, Inc. is a startup company whose business model is to compete against Amazon, but on a local scale. It requires its drivers, classified as "independent contractors," to maintain insurance on their personal vehicles, which the drivers are required to use to deliver packages. You are the most-recent hire in the Legal Department at Nile Delivers, Inc. Two other employees, both college-aged delivery drivers, started the same day. A week passes and one of the new drivers approaches you in the break room, flustered over something that just happened. Before you can even say "good morning," the driver volunteers that on her way to work, she hit a parked car while checking her cell phone, got flustered, and drove off. To make matters worse, she said she does not have auto insurance and asks your advice about whether she should obtain it immediately so she doesn't "get fired." Based on the week's materials, which among the following is the most-ethical path you can take? A. Tell the driver that because she is an independent contractor and the accident happened on the way to work, you would be happy to help, but for $250 an hour. B. Stop the conversation, leave the break room immediately, and ask your supervisor to put you behind an ethical wall. C. Tell the delivery driver you are the company's attorney and and tell her she's probably going to be fired. D. Tell the delivery driver you are the company's attorney, and advise her she might need to hire a lawyer.

D. Tell the delivery driver you are the company's attorney, and advise her she might need to hire a lawyer.

After taking countless multiple-choice questions, you have passed the MPRE and are focusing on other curriculum to finish up law school. You take a break from your studies, check social media, and see a video of a news conference of a criminal prosecutor addressing the news media. The prosecutor has a grim look on her face and is making a statement about a crime that occurred overnight. From a legal-ethics perspective, which of the prosecutor's statements below would cause you the most concern about the prosecutor's ethics -- even after spending several months not thinking much about the Model Rules of Professional Conduct, the comments, assigned reading, and office hours? A. The suspect is at large. B. The suspect is considered armed and dangerous. C. The suspect is a lawyer. D. The suspect is a "career criminal" because she has been convicted for doing this before.

D. The suspect is a "career criminal" because she has been convicted for doing this before.

True or false: According to our guest speakers from the University of Minnesota-Duluth, the trend toward "unbundling" legal services is a good idea likely to increase the quality of legal services provided to persons who live in rural communities.

False

True or false: A criminal prosecutor owes fiduciary duties only to innocent people.

False

True or false: A lawyer can be subject to discipline only for acts and omissions that arise in the state where the lawyer is licensed.

False

True or false: A lawyer has an ethical obligation to report another lawyer's misconduct, but only if the lawyers in question are (1) on opposing sides of pending or completed litigation or (2) attorneys at the same law firm or its equivalent.

False

True or false: Because law practice is a self-governing profession, the highest court in a state generally defines the practice of law and has the final say over whether a lawyer has engaged in unethical conduct.

True

True or false: A lawyer might have an ethical obligation to maintain confidentiality of information relating to the representation of a client even if the information is available in a public record.

True

True or false: Based on the week's office hour, it is highly advisable to put fee agreements in writing even when the Model Rules of Professional Conduct do not specifically require doing so.

True

When completing his bar application, Aaron was asked to disclose his record of all traffic violations. In his files, he found copies of two speeding tickets he received when he was 18 and he disclosed those. Regarding a speeding citation he received at age 16, he vividly remembers the traffic stop and the officer handing him a citation, but he could not find the documentation. Therefore, Aaron did not write anything about it on his bar application, which he signed under oath as being an accurate reflection of his infractions. Is Aaron subject to discipline? Yes, because Aaron misstated a material fact. Yes, because speeding is relevant to Aaron's character and fitness to practice law. No, because without the documentation, Aaron's recollection would be hearsay and hearsay is inadmissible evidence. No, because speeding tickets received before age 18 are civil-enforcement matters handled in juvenile court, not criminal court, so the rule of lenity applies.

Yes, because Aaron misstated a material fact.


Conjuntos de estudio relacionados

Skyler wanted me to make flashcards so here we are

View Set

Gov 100 Chapter 2 Learning Assessment

View Set

Weather and Climatology Chapter 9

View Set

Barbri - Practice Set Questions - MBE

View Set

Chapter 11 human anatomy and physiology

View Set